Đến nội dung

wallunint nội dung

Có 276 mục bởi wallunint (Tìm giới hạn từ 06-06-2020)



Sắp theo                Sắp xếp  

#275777 Ứng dụng của hoán vị !

Đã gửi bởi wallunint on 09-09-2011 - 13:59 trong Bất đẳng thức - Cực trị

Sắp phải kết thúc những tháng ngày bay bổng rồi zz

Mình mới tìm được một kĩ thuật đánh giá mới bằng p,q,r để giải bất đẳng thức hoán vị.

Mời các bạn chứng minh thử các bài toán này nhé :icon1:


Bài 1: Cho các số thực dương $a,b,c$. Chứng minh rằng:
$\dfrac{a}{{b\left( {3a + c} \right)}} + \dfrac{b}{{c\left( {3b + a} \right)}} + \dfrac{c}{{a\left( {3c + b} \right)}} \geqslant \dfrac{9}{{4\left( {a + b + c} \right)}}$

Bài 2: Cho các số thực dương $a,b,c$. Chứng minh rằng:
$\dfrac{{{a^3}}}{{b\left( {3a + c} \right)}} + \dfrac{{{b^3}}}{{c\left( {3b + a} \right)}} + \dfrac{{{c^3}}}{{a\left( {3c + b} \right)}} \geqslant \dfrac{{a + b + c}}{4}$

Bài 3: Cho các số thực dương $a,b,c$. Chứng minh rằng:
$\dfrac{{{a^3}b}}{{{a^2}b + 1}} + \dfrac{{{b^3}c}}{{{b^2}c + 1}} + \dfrac{{{c^3}a}}{{{c^2}a + 1}} \leqslant \dfrac{8}{9}.\dfrac{{\left( {a + b + c} \right)\left( {{a^3}b + {b^3}c + {c^3}a} \right)}}{{\sqrt[3]{{abc}}\left( {a + 1} \right)\left( {b + 1} \right)\left( {c + 1} \right)}}$


....... Còn nữa zz

Kĩ thuật này sẽ được trình bài đầy đủ trong một bài viết sắp tới của mình zz

Các bài toán trên mình tự sáng tác và chỉ giải được bằng p,q,r hoán vị.
Mong các bạn đóng góp những lời giải khác cho các bài toán trên



#273083 Thi IMO

Đã gửi bởi wallunint on 19-08-2011 - 13:08 trong Góc giao lưu

Chào mọi người. Cho em hỏi, em năm tới lên lớp 8, sau này muốn được thi IMO thì giờ cần học những phần nào? Hiện tại em đã đọc qua qua hết bộ sách GK toán đến lớp 8 đến lớp 12. Thanks

Em giỏi thật đấy zz Hồi anh học lớp 8 thầy cũng bảo anh đọc trước sách giáo khoa nhưng mà ko có đọc đến lớp 12 như em :D)

Phục em thật đấy! Nhưng theo anh nghĩ, em nên nghe lời anh perfectstrong thì hơn em ạ :mellow: Học kĩ thì có lẽ tốt hơn em ạ :D

Như anh Lê Hữu Phước (Đà Nẵng) thủ khoa TST năm vừa rồi :Rightarrow Lớp 8,9,10 là năm đổ bêtông. Sau đó dành thời gian bay bổng vào năm 11,12.

Tuy chỉ được huy chương đồng trong kì IMO vừa rồi nhưng chiến dịch của anh ấy thì không tệ đâu em ạ :huh:

Theo anh nghĩ, trào lưu của IMO hay cả TST hiện nay là đánh vào các điểm yếu học sinh như:
+ Số học
+ Tổ hợp (đặc biệt là hình học tổ hợp)

Như em thấy thì bài hình tổ hợp năm nay đoàn Việt Nam rất ít người làm đc ;) Nếu có ước mơ IMO thì em nên học kĩ và rèn luyện các vấn đề này :D

Mà em tên gì ấy nhỉ ?:D Để năm 2015,2016 anh tìm tên em trong đội tuyển Việt Nam cho nó dễ :D



#272933 Bất đẳng thức hay

Đã gửi bởi wallunint on 18-08-2011 - 12:58 trong Bất đẳng thức - Cực trị

Cho $a,b,c$ là các số thực thỏa mãn: $a^2+b^2+c^2+abc = 4$. Tìm giá trị lớn nhất của: $P=a+b+c$
Bài toán này tôi sưu tâm bên math.vn đây là bài toán có thể dễ dàng giải bằng phương pháp phản chứng nhưng còn có những cách làm hay khác mà không dựa vào bài toán ban đầu.
Nguuồn:http://math.vn/showthread.php?p=63550#post63550

Anh alex_hoang cho em xin lời giải bằng phản chứng của anh đc ko ạ ;)

Điều kiện của bài này khá thú vị. Tuy nhiên, ta có thể nới rộng nó một tí zz
Cho $a,b,c$ là các số thực thỏa mãn: $a^2+b^2+c^2+abc \leqslant 4$.
Tìm giá trị lớn nhất của: $P=a+b+c$
Lời giải:

Trong 3 số $1-a$, $1-b$, $1-c$ sẻ có 2 số cùng âm hoặc cùng dương. Giả sử đó là $1-a$ và $1-b$, ta được:

$\left( {1 - a} \right)\left( {1 - b} \right) \geqslant 0 \Leftrightarrow a + b \leqslant 1 + ab$

Kết hợp với điều kiện, ta được:

$\begin{gathered}4 \geqslant {a^2} + {b^2} + {c^2} + abc \geqslant 2ab + {c^2} + abc \hfill \\\Leftrightarrow 4 - {c^2} \geqslant ab\left( {2 + c} \right) \hfill \\\Leftrightarrow 2 - c \geqslant ab \hfill \\ \end{gathered} $

Từ đó, ta có:

$\begin{gathered}{\left( {a + b + c} \right)^2} \hfill \\= {a^2} + {b^2} + {c^2} + 2ab + 2bc + 2ca \hfill \\\leqslant 4 - abc + 4 - 2c + 2c\left( {a + b} \right) \hfill \\\leqslant 4 - abc + 4 - 2c + 2c\left( {1 + ab} \right) \hfill \\= 8 + abc \leqslant 9 \hfill \\ \end{gathered} $

Vậy bất đẳng thức đã được chứng minh.

ZZ



#272930 Mọi người xem giúp bài bđt !

Đã gửi bởi wallunint on 18-08-2011 - 12:40 trong Bất đẳng thức và cực trị

CMR với x,y,z là các số thực dương sao cho $ xyz=1$
$ \dfrac{1}{1+x^2}+\dfrac{1}{1+y^2}+\dfrac{1}{1+z^2}+\dfrac{2}{(1+x)(1+y)(1+z)}$ ;) $ 1 $

Đề của bạn zkobez hơi lạ đó à zz
Nếu đúng thì để phải thế này zz

Chứng minh rằng với x,y,z là các số thực dương sao cho $ xyz=1$
$ \dfrac{1}{(1+x)^2}+\dfrac{1}{(1+y)^2}+\dfrac{1}{(1+z)^2}+\dfrac{2}{(1+x)(1+y)(1+z)}$ :D $ 1 $



#272504 Một bài toán cũ và những bài toán mới

Đã gửi bởi wallunint on 15-08-2011 - 13:53 trong Bất đẳng thức và cực trị

Ở một topic cũ, bạn tangkhaihanh có post một bài toán trong THTT số 367 như sau:

Cho $a, b, c$ là các số thực không âm. Chứng minh rằng
$(a^2+b^2+c^2)^2 \geq 4(a-b)(b-c)(c-a)(a+b+c)$

Lời giải mình đã post ở đây: http://diendantoanho...showtopic=58051

Bất đẳng thức trên khá đẹp và khó. Và sau bất đẳng thức này, ta còn có thể có nhiều khám phá rất thú vị.
Các bạn hãy thử chứng minh một số bài toán sau:

Bài1: (Trần Tuấn Anh) Cho $a, b, c$ là các số thực không âm thỏa mãn $a,b,c \in \left[ {0,1} \right]$. Tìm max của:
$4\left( {a - b} \right)\left( {b - c} \right)\left( {c - a} \right)\left( {a + b + c} \right)$

Bài 2: (Wallunint) Cho các số thực không âm $a,b,c$. Chứng minh rằng:
$\sum {{a^4}} - abc\left( {\sum a } \right) \geqslant 4\left( {a - b} \right)\left( {b - c} \right)\left( {c - a} \right)\left( {a + b + c} \right)$

Bài toán này còn có nhiều mở rộng thú vị. Mình sẽ tiếp tục đưa các mở rộng khác vào lần sau.
Mong các bạn có những khám phá thú vị từ những bất đẳng thức này.


Có ai giải mấy bài này của mình ko nhỉ :sqrt{a}

Đây là một bài tương tự!
Bài 3: (IMO 2006)Cho $a, b, c$ là các số thực không âm thỏa mãn $a^2+b^2+c^2$. Tìm max của:
$P=\left( {a - b} \right)\left( {b - c} \right)\left( {c - a} \right)\left( {a + b + c} \right)$
Theo gợi ý của wallunint mình chém bài 3 theo AM-GM:
Ta có:
$[3(a^2+b^2+c^2)]^2 =[2(a-b)^2+2(a-c)(b-c)+(a+b+c)^2]^2 \geq$
$ \geq 8|(a-c)(b-c)|[2(a-b)^2+(a+b+c)^2]$
$ \geq 16\sqrt{2}\left( {a - b} \right)\left( {b - c} \right)\left( {c - a} \right)\left( {a + b + c} \right)|$
$ \Rightarrow P \leq \dfrac{9}{16\sqrt{2}}$


Thanks bboy114crew !!! Lâu rồi mới vào lại cái topic này zz Cách giải khác cho bài 3:

Chia 2 trường hợp: $a \geqslant b \geqslant c$ Trường hợp 2: $a \leqslant b \leqslant c$

Áp dụng bất đẳng thức AM-GM, ta được:
$\left| {\left( {a - b} \right)\left( {b - c} \right)} \right| = \left( {b - a} \right)\left( {c - b} \right) \leqslant {\left( {\dfrac{{b - a + c - b}}{2}} \right)^2} \leqslant \dfrac{{{{\left( {a - c} \right)}^2}}}{4}$
Ta lại có: ${\left( {\dfrac{{\left( {b - a} \right) + \left( {c - b} \right)}}{2}} \right)^2} \leqslant \dfrac{{{{\left( {c - b} \right)}^2} + {{\left( {b - a} \right)}^2}}}{4}$
Từ đó, ta có: $3{\left( {c - a} \right)^2} \leqslant 2\left[ {{{\left( {b - a} \right)}^2} + {{\left( {c - b} \right)}^2} + {{\left( {c - a} \right)}^2}} \right]$
Tiếp tục áp dụng bất đẳng thức AM-GM, ta được:
$P \leqslant \dfrac{1}{4}\sqrt {{{\left( {c - a} \right)}^6}{{\left( {a + b + c} \right)}^2}} \leqslant \dfrac{1}{{\sqrt 2 }}\left( {\sqrt[4]{{{{\left( {\dfrac{{\left[ {{{\left( {b - a} \right)}^2} + {{\left( {c - b} \right)}^2} + {{\left( {c - a} \right)}^2}} \right]}}{3}} \right)}^3}{{\left( {a + b + c} \right)}^2}}}} \right)$
Từ đó, ta có:
$\begin{gathered}P \leqslant \dfrac{1}{{\sqrt 2 }}{\left( {\dfrac{{{{\left( {b - a} \right)}^2} + {{\left( {c - b} \right)}^2} + {{\left( {c - a} \right)}^2} + {{\left( {a + b + c} \right)}^2}}}{4}} \right)^2} \hfill \\\Leftrightarrow P \leqslant \dfrac{9}{{16\sqrt 2 }}{\left( {{a^2} + {b^2} + {c^2}} \right)^2} \hfill \\ \end{gathered} $

Ps: Cách này tuy hơi dài, nhưng dễ dàng hơn trong việc chọn điểm rơi !!!
Các bạn hãy thử giải bất đẳng thức này bằng Holder !!!

ZZ



#272499 Topic về Bất đẳng thức, cực trị THCS

Đã gửi bởi wallunint on 15-08-2011 - 12:35 trong Bất đẳng thức và cực trị

Bài 25: Cho x, y là các số thực không âm thỏa mãn $x^3 + y^3 = 1$. Tìm GTLN của:

$P = \sqrt x + 2\sqrt y $.


Bài này chưa có lời giải zz Có ai giải đc ko zz
Gợi ý:
+ Sử dụng bất đẳng thức $\left( {{a^3} + {b^3}} \right)\left( {{x^3} + {y^3}} \right)\left( {{m^3} + {n^3}} \right) \geqslant {\left( {{axm + byn}} \right)^3}$
+ Từ đó, ta cần chứng minh: $\sqrt x + 2\sqrt y \leqslant \sqrt[6]{{{{\left( {1 + 2\sqrt[5]{2}} \right)}^5}}}$

Chú lớp 8 mà khiếp!
Bài 20 Cho các số$a,b,c \geq 0$ .CMR:
$(\dfrac{a}{a+b})^3+(\dfrac{b}{c+b})^3+(\dfrac{c}{a+c})^3 \geq \dfrac{3}{8}$


Đây là đề thi VMO zz
Cách giải của bạn Nguyễn Hữu Huy hình như sai zz Cẩn thận cái vai trò của các biến nhé zz
Có 1 bài toán mạnh hơn như sau:

Bài 29: Cho các số$a,b,c \geq 0$ .CMR:
${\left( {\dfrac{a}{{a + b}}} \right)^3} + {\left( {\dfrac{b}{{c + b}}} \right)^3} + {\left( {\dfrac{c}{{a + c}}} \right)^3} + \dfrac{{5abc}}{{\left( {a + b} \right)\left( {b + c} \right)\left( {c + a} \right)}} \geqslant 1$


Giúp em bài này cái
Bài 21
Chứng minh bất đẳng thức
$\dfrac{ a^{3}}{a^{2}+ab+ b^{2}}+\dfrac{ b^{3}}{b^{2}+bc+c^{2}}+\dfrac{c^{3}}{c^{2}+ca+a^{2}}\geq \dfrac{a+b+c}{3}$


Bạn thử chứng minh bài toán chặt hơn như sau:

Bài 30: Cho các số$a,b,c \geq 0$ .CMR:
$\dfrac{{{a^3}}}{{{a^3} + abc + {b^3}}} + \dfrac{{{b^3}}}{{{b^3} + abc + {c^3}}} + \dfrac{{{c^3}}}{{{c^3} + abc + {a^3}}} \geqslant \dfrac{{{a^3} + {b^3} + {c^3}}}{{{a^2} + {b^2} + {c^2}}}$
Vì sao bất đẳng thức này lại chặt hơn ??? Các bạn tự chứng minh nhé zz


ps xusinst @: How old are you ??

ZZ



#272475 Tính số tam giác tạo bởi 3 đường chéo (Tổ hợp hay)

Đã gửi bởi wallunint on 15-08-2011 - 10:29 trong Tổ hợp - Xác suất và thống kê - Số phức

Bài này bạn perfectstrong nên về suy nghĩ lại là đc thôi !!!
Bài này trên lớp A2 có làm 1 lần rồi !!!

Đề chuẩn: Cho tứ giác lồi có n cạnh. Hỏi có bao nhiêu tam giác được tạo thành từ các đường chéo ko tự cắt nhau ??

ps ongtroi @: Bài này mà đa giác lỏm sao làm được anh :sqrt{a}

ZZ



#270252 Bất đẳng thức

Đã gửi bởi wallunint on 30-07-2011 - 20:56 trong Bất đẳng thức và cực trị

Mình viết lại bài toán trên ở dạng đơn giản và theo mình là dễ dàng hơn nhiều các bạn chém thử
Cho các số thực không âm $x,y,z$ sao cho $x+y+z=3$.CMR
$2 + xyz \ge xy + yz + zx$

thì ra bài này có ở đây
http://www.artofprob...v...51&t=420085

Chào các bạn :D
Bài toán này có thể dễ dàng áp dụng bất đẳng thức Schur để chứng minh :D Mời các bạn thử chứng minh bài toán khá giống như sau:
Cho các số thực không âm $x,y,z$ sao cho $x^2+y^2+z^2=3$.CMR
$2 + xyz \ge xy + yz + zx$



#268829 Bất đẳng thức trong so THTT so 367

Đã gửi bởi wallunint on 17-07-2011 - 18:39 trong Bất đẳng thức và cực trị

Thực ra vai trò của các biến trong bất đẳng thức này không như nhau đâu :delta
Vai trò của các biến trong bài toán này có tính hoán vị đấy :delta Lên lớp 10 học sẽ thấy rõ hơn thôi :delta

ps: sao ai cũng vik sai tên mình thế nhỉ :delta( Wallunint mới đúng chứ :delta(



#268698 Chuyên đề về phương trình bậc hai

Đã gửi bởi wallunint on 16-07-2011 - 10:25 trong Đại số

Bài 33 Cho $x,y$ thỏa mãn:
${x^2} + 2ax + 9 = 0$ với $a \geq 3$
${y^2} + 2by + 9 = 0$ với $b \geq 3$
Tìm giá trị nhỏ nhất của: $A = {\left( {x - y} \right)^2} + 3{\left( {\dfrac{1}{x} - \dfrac{1}{y}} \right)^2}$
Giải :
Ta có : $ A = {\left( {x - y} \right)^2} + 3{\left( {\dfrac{1}{x} - \dfrac{1}{y}} \right)^2} \geq 0 $
Vậy $ min_A = 0 $ khi : $ x = y ; \dfrac{1}{x} = \dfrac{1}{y} \Rightarrow x = y \neq 0 $
Với $ x = y$, hai phương trình trên có thể đưa được về dạng :
$ x^2 + 2ax + 9 = 0 $
$ x^2 + 2bx + 9 = 0 $
Trừ vế theo vế của hai phương trình, ta có : $ ( x^2 + 2ax+ 9 ) - ( x^2 + 2bx + 9) = 0$
$ \Leftrightarrow 2ax - 2bx = 0 \Leftrightarrow 2x( a - b ) = 0 $
Mặt khác : $ x \neq 0 $. Do đó a = b.
Vậy $ A = {\left( {x - y} \right)^2} + 3{\left( {\dfrac{1}{x} - \dfrac{1}{y}} \right)^2}$ nhỏ nhất khi $ a = b \geq 3$
P/S : Không biết sao nữa ! Cho nhận xét đi ! À mà nhớ đánh số thứ tự với kìa GS.

Bài này mình mới sửa lại đề :x Bạn coi lại đi nhá :x
Bài này giải thế là hỏng rồi :Leftrightarrow Vì khi thế $a$ và $b$ vào hai phương trính trên ta không tính được giá trị của $x$ và $y$ :Leftrightarrow

Ta cần chú ý rằng $x$ và $y$ trái dấu :perp
Và ${A_{\min }} = 8\sqrt 3 \Leftrightarrow x = - \dfrac{1}{{\sqrt[4]{3}}};y = \dfrac{1}{{\sqrt[4]{3}}}$



#268651 Chuyên đề về phương trình bậc hai

Đã gửi bởi wallunint on 15-07-2011 - 22:18 trong Đại số

Có một số bài toán cũ nhưng khá hay :x Các bạn hãy làm thử :Leftrightarrow

Bài 33: Cho $x,y$ thỏa mãn:
${x^2} + 2ax + 9 = 0$ với $a \geqslant 3$
${y^2} - 2by + 9 = 0$ với $b \geqslant 3$
Tìm giá trị nhỏ nhất của: $A = {\left( {x - y} \right)^2} + 3{\left( {\dfrac{1}{x} - \dfrac{1}{y}} \right)^2}$

Bài 34: Tìm số hữu tỉ $p$ sao cho pt sau có ít nhất 1 nghiệm nguyên:
${x^2} - 2\left( {p - 1} \right)x + {p^2} - 6p + 11 = 0$

ps: Hic hic :x Ghi nhầm đề, mong các bạn thông cảm!!



#268569 Trại hè toán học 2011

Đã gửi bởi wallunint on 15-07-2011 - 09:14 trong Thông báo tổng quan

Trại hè gì vậy Tường? sao mình không biết :Leftrightarrow

Kinh phí năm nay hình như hơi đắt thì phải :Leftrightarrow
Nghe nói phải hơn 2 triệu :perp(
Chưa kể tiền ăn, tiền thuê khách sạn,..., tiền tùm lum :in(
Sợ năm nay ở nhà nằm ngủ thôi :in
Để mấy bác có điều kiện đi ra đó chơi :in
Mà nghe anh Long nói cuối kì còn có đợt kiểm tra nữa :x)
Kiểm tra mà ko đc gì nhục lắm :x



#268487 Một bài toán cũ và những bài toán mới

Đã gửi bởi wallunint on 14-07-2011 - 13:51 trong Bất đẳng thức và cực trị

Ở một topic cũ, bạn tangkhaihanh có post một bài toán trong THTT số 367 như sau:

Cho $a, b, c$ là các số thực không âm. Chứng minh rằng
$(a^2+b^2+c^2)^2 \geq 4(a-b)(b-c)(c-a)(a+b+c)$

Lời giải mình đã post ở đây: http://diendantoanho...showtopic=58051

Bất đẳng thức trên khá đẹp và khó. Và sau bất đẳng thức này, ta còn có thể có nhiều khám phá rất thú vị.
Các bạn hãy thử chứng minh một số bài toán sau:

Bài1: (Trần Tuấn Anh) Cho $a, b, c$ là các số thực không âm thỏa mãn $a,b,c \in \left[ {0,1} \right]$. Tìm max của:
$4\left( {a - b} \right)\left( {b - c} \right)\left( {c - a} \right)\left( {a + b + c} \right)$

Bài 2: (Wallunint) Cho các số thực không âm $a,b,c$. Chứng minh rằng:
$\sum {{a^4}} - abc\left( {\sum a } \right) \geqslant 4\left( {a - b} \right)\left( {b - c} \right)\left( {c - a} \right)\left( {a + b + c} \right)$

Bài toán này còn có nhiều mở rộng thú vị. Mình sẽ tiếp tục đưa các mở rộng khác vào lần sau.
Mong các bạn có những khám phá thú vị từ những bất đẳng thức này.



#268484 Trại hè toán học 2011

Đã gửi bởi wallunint on 14-07-2011 - 13:20 trong Thông báo tổng quan

Năm nay, có lẽ ở Lê Quý Đôn cũng cho đi trại hè :D. Vì vậy chắc không cần đăng kí.
Nhưng mà năm nay thì mình chắc cũng được đi rồi :-? (Mặc dù điều kiện ko được tốt lắm)
Ai đi thì pm cho mình nhá.



#262306 My Inequality

Đã gửi bởi wallunint on 27-05-2011 - 17:54 trong Bất đẳng thức và cực trị

bài này cũ rồi mà bạn.
Cách đơn giản nhất là đặt a = yz/x^2
b = xz/y^2
c = xy/z^2
rồi swarchs bình thường là xong luôn

Còn cách đặt nào nữa ko ?? ^_^
Bạn giải rõ ra đi :D



#262272 My Inequality

Đã gửi bởi wallunint on 27-05-2011 - 12:30 trong Bất đẳng thức và cực trị

Chào các bạn :in
Mình mới chế 1 bất đẳng thức và đã tìm đc 3 lời giải cho nó. Mong các bạn góp ý cho :vdots
Mình cũng tìm đc một phương pháp mới để chứng minh bất đẳng thức và ta cùng tìm hiểu nó qua bất đẳng thức này nhé :vdots

Cho các số thực dương $a,b,c$ thỏa mãn$abc=1$. CMR:

$\dfrac{1}{{{{\left( {1 + a} \right)}^2}}} + \dfrac{1}{{{{\left( {1 + b} \right)}^2}}} + \dfrac{1}{{{{\left( {1 + c} \right)}^2}}} \geqslant \dfrac{3}{4}$





#261720 Topic về bất đẳng thức

Đã gửi bởi wallunint on 22-05-2011 - 13:16 trong Bất đẳng thức - Cực trị

Tiếp tục nào :D
Bài 9: Cho các số thực không âm $a,b,c$ thỏa mãn$a+b+c=2$. CMR:

$\dfrac{{bc}}{{{a^2} + 1}} + \dfrac{{ac}}{{{b^2} + 1}} + \dfrac{{ab}}{{{c^2} + 1}} \leqslant 1$


*Nhận xét : Bài này có khá nhiều cách giải rất trâu bò
Các bạn có thể tìm đc bao nhiêu cách giải cho bài này ??? :D


Bài 2: Cho các số thực không âm $a,b,c$ sao cho không có 2 số nào đồng thời bằng $0$. CMR:

$\sqrt {\dfrac{{{a^3}}}{{{a^3} + {{\left( {b + c} \right)}^3}}}} + \sqrt {\dfrac{{{b^3}}}{{{b^3} + {{\left( {a + c} \right)}^3}}}} + \sqrt {\dfrac{{{c^3}}}{{{c^3} + {{\left( {a + b} \right)}^3}}}} \geqslant 1$

Các bạn nhanh chóng giải bài này đi :D
Gợi ý : Chỉ sử dụng bất đẳng thức $AM-GM$ $(Cauchy)$

Đừng post bài nữa nhá :D để giải xong mấy bài trên rồi post tiếp nha :D



#261712 Topic về bất đẳng thức

Đã gửi bởi wallunint on 22-05-2011 - 12:09 trong Bất đẳng thức - Cực trị

Bài 7: Cho $a, b, c$ là ba số thực dương tùy ý. Chứng minh rằng:

$\sum\limits_{cyc} \sqrt{\dfrac{b + c}{a}} \ge 2\left(\sum\limits_{cyc} \sqrt{\dfrac{a}{b + c}}\right).\sqrt{1 + \dfrac{(a + b)(b + c)(c + a) - 8abc}{4\sum\limits_{cyc} a(a + b)(a + c)}}.$


Các bạn xem lời giải của mình ở đây :D

Solution



#261589 Topic về bất đẳng thức

Đã gửi bởi wallunint on 21-05-2011 - 11:41 trong Bất đẳng thức - Cực trị



2) Cho các số thực không âm $a,b,c$ sao cho không có 2 số nào đ�ồng thời bằng $0$. CMR:

$\sqrt {\dfrac{{{a^3}}}{{{a^3} + {{\left( {b + c} \right)}^3}}}} + \sqrt {\dfrac{{{b^3}}}{{{b^3} + {{\left( {a + c} \right)}^3}}}} + \sqrt {\dfrac{{{c^3}}}{{{c^3} + {{\left( {a + b} \right)}^3}}}} \geqslant 1$


Lần sau anh dark templar và anh h.vuong_pdl giải rõ ra nhá :D Không thì em xóa bài đấy :D
2 cách giải sơ cấp cho bài này như sau :D
Cách 1: Chứng minh $\sqrt {\dfrac{{{a^3}}}{{{a^3} + {{(b + c)}^3}}}} \geqslant \dfrac{{{a^2}}}{{{a^2} + {b^2} + {c^2}}}$

$ \Leftrightarrow 2{a^2}\left( {{b^2} + {c^2}} \right) + {\left( {{b^2} + {c^2}} \right)^2} \geqslant a{\left( {b + c} \right)^3}$

Áp dụng bất đẳng thức Cauchy-Schwarz, ta có:

$2\left( {{b^2} + {c^2}} \right) \geqslant {\left( {b + c} \right)^2} \Leftrightarrow {\text{8}}{\left( {{b^2} + {c^2}} \right)^3} \geqslant {\left( {b + c} \right)^6}$

Tiếp tục áp dụng bất đẳng thức AM-GM, ta có:

${a^2}\left( {{b^2} + {c^2}} \right) + {\left( {{b^2} + {c^2}} \right)^2} \geqslant 2\sqrt {{a^2}{{\left( {{b^2} + {c^2}} \right)}^3}} \geqslant a{\left( {b + c} \right)^3}$

Vậy, phép chứng minh hoàn tất. :D

Cách 2: sử dụng bất đẳng thức AM-GM và viết lại bất đẳng thức như sau :



ps: đi học về post tiếp :D



#261478 Topic về bất đẳng thức

Đã gửi bởi wallunint on 20-05-2011 - 13:03 trong Bất đẳng thức - Cực trị

Topic về bất đẳng thức


1) Lời nói đầu :
Bất đẳng thức là một chủ đề khá thú vị trên các diển đàn hiện nay với một số lượng bài viết khá lớn. Nhưng ở VMF, số bài viết còn chưa nhiều và còn quá loảng, chủ yếu tập trung ở forum toán cấp 2 thôi :D Mình lấp topic này mong các bạn nhiệt tình tham gia ^_^ Qua topic này, mong rằng có thể cùng các bạn thảo luận và khám phá nhiều bất đẳng thức mới.

2) Quy định post bài :
+ Chỉ được post 2 bài 1 lần. Giải xong mới được post tiếp để tránh hiện tượng Spam và loãng topic :D Nếu bài khó quá thì để lại từ từ giải, chuyển qua bài khác.
+ Không Spam
+ Bài giải phải đầy đủ các bước ( nói tóm tắt cũng đc).
+ Dùng từ ngữ đúng theo ngữ pháp Tiếng Việt. Và phải dùng đúng latex.
+ Bài viết vi phạm các quy định trên thì sẽ bị xóa không thương tiếc ^_^


Mở mình xin mở đầu topic bằng 2 bài toán sau ^_^

Bài 1: Cho các số thực không âm $a,b,c$ sao cho không có 2 số nào đồng thời bằng $0$. CMR:

$\sqrt {\dfrac{{{a^3}}}{{{a^2} + ab + {b^2}}}} + \sqrt {\dfrac{{{b^3}}}{{{b^2} + bc + {c^2}}}} + \sqrt {\dfrac{{{c^3}}}{{{c^2} + ac + {a^2}}}} \geqslant \dfrac{{\sqrt a + \sqrt b + \sqrt c }}{{\sqrt 3 }}$


Bài 2: Cho các số thực không âm $a,b,c$ sao cho không có 2 số nào đồng thời bằng $0$. CMR:

$\sqrt {\dfrac{{{a^3}}}{{{a^3} + {{\left( {b + c} \right)}^3}}}} + \sqrt {\dfrac{{{b^3}}}{{{b^3} + {{\left( {a + c} \right)}^3}}}} + \sqrt {\dfrac{{{c^3}}}{{{c^3} + {{\left( {a + b} \right)}^3}}}} \geqslant 1$




#261082 Bất đẳng thức trong so THTT so 367

Đã gửi bởi wallunint on 16-05-2011 - 22:56 trong Bất đẳng thức và cực trị

Cho a, b, c là các số thực không âm. Chứng minh rằng
$(a^2+b^2+c^2)^2 \geq 4(a-b)(b-c)(c-a)(a+b+c)$
Giúp em với em chưa làm được

À :D Bài này thì ko khó cho lắm :D Thực ra mình đã có chứng minh 1 bài tương tự rồi :D

Thực ra suy nghĩ của bạn khanh3570883 cũng là 1 phần của bài toán trên.

Giả sử $a \geqslant b \geqslant c$ thì bất đẳng thức luôn đúng.

Xét $a \leqslant b \leqslant c $. Áp dụng bất đăng thức AM-GM, ta có:
$4\left( {a - b} \right)\left( {b - c} \right)\left( {c - a} \right)\left( {a + b + c} \right) \leqslant {\left[ {\left( {x + y + z} \right)\left( {y - x} \right) + \left( {z - y} \right)\left( {z - x} \right)} \right]^2}$

Vì vậy, bất đăng thức cần chứng minh tương đương với:

${a^2} + {b^2} + {c^2} \geqslant \left( {x + y + z} \right)\left( {y - x} \right) + \left( {z - y} \right)\left( {z - x} \right)$
$\begin{gathered}\Leftrightarrow x\left( {2x + 2z - y} \right) \geqslant 0:luon\_dung \hfill \\\Rightarrow \left( {dpcm} \right) \hfill \\\end{gathered} $


ps: Trong này ko bit ai nhiều tuổi hơn nên cứ gọi là bạn vậy :D



#260705 Những bất đẳng thức hay !

Đã gửi bởi wallunint on 08-05-2011 - 12:48 trong Bất đẳng thức và cực trị

Bài số 2 đầu topic này có thể giải bằng phương pháp p,q,r hoặc sử dụng bất đẳng thức Schur :)
Một bài khác cũng gần giống, nhưng khó chịu 1 tí :)

Cho các số thực dương $a,b,c$. CMR:
$\dfrac{{{a^2}}}{{{a^2} + ab + {b^2}}} + \dfrac{{{b^2}}}{{{c^2} + cb + {b^2}}} + \dfrac{{{c^2}}}{{{a^2} + ac + {c^2}}} \geqslant 1$




ps : anh qua Bài trên không cần phải dồn biến thừa trừ đâu nhá :D Còn cách khác bằng Cauchy-Schwraz đấy :D



#260702 Một bất đẳng thức đẹp - IMO 2005

Đã gửi bởi wallunint on 08-05-2011 - 12:37 trong Bất đẳng thức và cực trị

Đề nghị các cao thủ giải bài này theo phương pháp của THCS thôi nhé :)
Cho a,b,c là các số thực dương thỏa mãn $abc \geq 1 $
Chứng minh rằng:
$\dfrac{a^5-a^2}{a^5+b^2+c^2}+\dfrac{b^5-b^2}{b^5+c^2+a^2}+\dfrac{c^5-c^2}{c^5+a^2+b^2} \geq 0 $
(IMO 2005)

Kĩ thuật giải bài này gần giống cách giải bài của anh Cường trong cuốn Cauchy-Schwarz :)
Bất đẳng thức cần chứng minh tương đương với :
$\sum {\dfrac{1}{{{x^5} + {y^2} + {z^2}}}} \geqslant \dfrac{3}{{{x^2} + {y^2} + {z^2}}}$
Áp dụng bất đẳng thức Cauchy-Schwarz, ta có :
$\left( {{x^5} + {y^2} + {z^2}} \right)\left( {yz + {y^2} + {z^2}} \right) \geqslant \left( {\sqrt {{x^5}yz} + {y^2} + {z^2}} \right) \geqslant \left( {{x^2} + {y^2} + {z^2}} \right)$
Vì vây, ta có :
$\sum {\dfrac{1}{{{x^5} + {y^2} + {z^2}}}} \leqslant \sum {\dfrac{{yz + {y^2} + {z^2}}}{{{x^2} + {y^2} + {z^2}}}} \leqslant \sum {\dfrac{{\dfrac{{{y^2} + {z^2}}}{2} + {y^2} + {z^2}}}{{{x^2} + {y^2} + {z^2}}}} = \dfrac{3}{{{x^2} + {y^2} + {z^2}}}$
Vậy, bất đẳng thức đã được chứng minh xong :D



#260556 Cùng chém bất đẳng thức

Đã gửi bởi wallunint on 07-05-2011 - 17:47 trong Bất đẳng thức và cực trị

Cho $x, y, z $ :D 1 thoả mãn $ xyz=1, CMR : $
${\left( {\dfrac{x}{{x - 1}}} \right)^2} + {\left( {\dfrac{y}{{y - 1}}} \right)^2} + {\left( {\dfrac{z}{{z - 1}}} \right)^2} \geqslant 1$
wallunint @ Đây là IMO năm 2008
Các bạn có thể xem bài viết của anh Nguyễn Đình thi trên bào Toán Tuối Thơ về bài tổng quát

Thực ra, bài này còn có 1 cách khác :D Các bạn thử góp ý nhá :leq
Đặt $x = \dfrac{{bc}}{{{a^2}}},y = \dfrac{{ca}}{{{b^2}}},z = \dfrac{{ab}}{{{c^2}}}$, bất đẳng thức cần chứng minh trở thành:
$\sum {\dfrac{{{a^4}}}{{{{({a^2} - bc)}^2}}}} \geqslant 1$
Đến đây, ta áp dụng bất đẳng thức Cauchy-Schwarz và ta áp dụng bổ đề sau :
$\sum {{{\left( {{a^2} - bc} \right)}^2}} \leqslant \sum {{{\left( {{a^2}} \right)}^2}} $
Vậy, bất đẳng thức đã được chứng minh xong.



#260531 Cực trị

Đã gửi bởi wallunint on 07-05-2011 - 16:31 trong Bất đẳng thức và cực trị


P/S : Đề là thế này hay là : Tìm giá trị lớn nhất của $ A = \dfrac{1}{\sqrt{x}.y}$ , khi X và Y thỏa mãn điều kiện: $ \dfrac{1}{\sqrt{x}} + \dfrac{1}{\sqrt{y}} = 6 $ .

Mình giải thế này, không biết thế nào :D
Áp dụng bất đẳng thức Cauchy, ta có :
$\begin{gathered}\dfrac{{27}}{{4\sqrt x \sqrt y \sqrt y }} \leqslant {\left( {\dfrac{1}{{\sqrt x }} + \dfrac{1}{{2\sqrt y }} + \dfrac{1}{{2\sqrt y }}} \right)^3} = 216 \hfill \\\Rightarrow \dfrac{1}{{\sqrt x y}} \leqslant 32 \hfill \\ \end{gathered} $